1
$\begingroup$

I would like to ask about a certain inequality that I need and which came out of some work in here.

Question. For integers $n\geq1$ and $k\geq3$, is this true? If so, any proof? $$6\prod_{j=1}^k(3n+j)\geq k!\,(nk+3)(nk+2)(nk+1).$$

Caveat. I'm not particularly interested in asymptotic analysis because for fixed $n$ it is rather clear what happens when $k$ is large.

$\endgroup$

2 Answers 2

3
$\begingroup$

For $n=1$ we have an equality, so assume that $n\geqslant 2$. For $k=3$ we have an equality, so induct on $k$ proving that $LHS/RHS$ increases if we replace $k$ to $k+1$. This is equivalent $$1+\frac{3n}{k+1}\geqslant \left(1+\frac{n}{nk+3}\right)\left(1+\frac{n}{nk+2}\right)\left(1+\frac{n}{nk+1}\right).$$ Each multiple on the right does not exceed $1+1/k$, thus it suffices to prove that $1+6/(k+1)\geqslant (1+1/k)^3$, that is true for $k\geqslant 3$.

$\endgroup$
0
$\begingroup$

Thanks, Fedor. I've found a proof using induction on $k$, for any $n\geq1$.

Obvious when $k=3$ (equality holds). Assume true for some $k$. The LHS for $k+1$ after induction: $$6\prod_{j=1}^{k+1}(3n+j)=(3n+k+1)6\prod_{j=1}^k(3n+j)\geq(3n+k+1)k!(nk+3)(nk+2)(nk+1).$$ It suffices to show that $(3n+k+1)k!\prod_{j=1}^3(nk+j)\geq (k+1)!\prod_{j=1}^3(nk+n+j)$; or, $(3n+k+1)\prod_{j=1}^3(nk+j)- (k+1)\prod_{j=1}^3(nk+n+j)\geq0$. After simplifying, this inequality takes the form: \begin{align*} n(n-1)(81n^2+95n+26)&+n(n-1)(81n^2+68n+11)(k-3) \\ &+3n^2(9n+4)n-1)(k-3)^2+3n^3(n-1)(k-3)^3\geq0. \end{align*} Now, this is evident when $k\geq3$ and $n\geq1$. The proof is complete.

$\endgroup$

Your Answer

By clicking “Post Your Answer”, you agree to our terms of service and acknowledge you have read our privacy policy.

Not the answer you're looking for? Browse other questions tagged or ask your own question.